Difference between revisions of "1954 AHSME Problems/Problem 37"

(Problem 37)
(Problem 37)
Line 45: Line 45:
 
<math>\textbf{(A)}\ \angle m = \frac {1}{2}(\angle p - \angle q) \qquad  
 
<math>\textbf{(A)}\ \angle m = \frac {1}{2}(\angle p - \angle q) \qquad  
 
\textbf{(B)}\ \angle m = \frac {1}{2}(\angle p + \angle q) </math>\qquad
 
\textbf{(B)}\ \angle m = \frac {1}{2}(\angle p + \angle q) </math>\qquad
<math> \textbf{(C)}\ \angle d =\frac{1}{2}(\angle q+\angle p)\qquad
+
<math> \textbf{(C)}\ \angle d =\frac{1}{2}(\angle q+\angle p)</math>\qquad<math>
 
\textbf{(D)}\ \angle d =\frac{1}{2}\angle m\qquad
 
\textbf{(D)}\ \angle d =\frac{1}{2}\angle m\qquad
 
\textbf{(E)}\ \text{none of these is correct} </math>
 
\textbf{(E)}\ \text{none of these is correct} </math>

Revision as of 20:43, 14 April 2016

Problem 37

Given $\triangle PQR$ with $\overline{RS}$ bisecting $\angle R$, $PQ$ extended to $D$ and $\angle n$ a right angle, then:

[asy] path anglemark2(pair A, pair B, pair C, real t=8, bool flip=false) {  pair M,N;  path mark;  M=t*0.03*unit(A-B)+B;  N=t*0.03*unit(C-B)+B;  if(flip)  mark=Arc(B,t*0.03,degrees(C-B)-360,degrees(A-B));  else  mark=Arc(B,t*0.03,degrees(A-B),degrees(C-B));  return mark; } unitsize(1.5cm); defaultpen(linewidth(.8pt)+fontsize(8pt)); pair P=(0,0), R=(3,2), Q=(4,0); pair S0=bisectorpoint(P,R,Q); pair Sp=extension(P,Q,S0,R); pair D0=bisectorpoint(R,Sp), Np=midpoint(R--Sp); pair D=extension(Np,D0,P,Q), M=extension(Np,D0,P,R); draw(P--R--Q); draw(R--Sp); draw(P--D--M); draw(anglemark2(Sp,P,R,17)); label("$p$",P+(0.35,0.1)); draw(anglemark2(R,Q,P,11)); label("$q$",Q+(-0.17,0.1)); draw(anglemark2(R,Np,D,8,true)); label("$n$",Np+(+0.12,0.07)); draw(anglemark2(R,M,D,13,true)); label("$m$",M+(+0.25,0.03)); draw(anglemark2(M,D,P,29)); label("$d$",D+(-0.75,0.095)); pen f=fontsize(10pt); label("$R$",R,N,f); label("$P$",P,S,f); label("$S$",Sp,S,f); label("$Q$",Q,S,f); label("$D$",D,S,f);[/asy]

$\textbf{(A)}\ \angle m = \frac {1}{2}(\angle p - \angle q) \qquad  \textbf{(B)}\ \angle m = \frac {1}{2}(\angle p + \angle q)$\qquad $\textbf{(C)}\ \angle d =\frac{1}{2}(\angle q+\angle p)$\qquad$\textbf{(D)}\ \angle d =\frac{1}{2}\angle m\qquad \textbf{(E)}\ \text{none of these is correct}$

Partial Solution

[asy] import math; path anglemark2(pair A, pair B, pair C, real t=8, bool flip=false) {  pair M,N;  path mark;  M=t*0.03*unit(A-B)+B;  N=t*0.03*unit(C-B)+B;  if(flip)  mark=Arc(B,t*0.03,degrees(C-B)-360,degrees(A-B));  else  mark=Arc(B,t*0.03,degrees(A-B),degrees(C-B));  return mark; } unitsize(1.5cm); defaultpen(linewidth(.8pt)+fontsize(8pt)); pair P=(0,0), R=(3,2), Q=(4,0); pair S0=bisectorpoint(P,R,Q); pair Sp=extension(P,Q,S0,R); pair D0=bisectorpoint(R,Sp), Np=midpoint(R--Sp); pair D=extension(Np,D0,P,Q), M=extension(Np,D0,P,R); draw(P--R--Q); draw(R--Sp); draw(P--D--M); pen f=fontsize(10pt); pair pI=extension(D,M,R,Q); label("$O$",pI+(-0.2,0.166),f); draw(anglemark2(Sp,P,R,17)); label("$p$",P+(0.35,0.1)); draw(anglemark2(R,Q,P,11)); label("$q$",Q+(-0.17,0.1)); draw(anglemark2(R,Np,D,8,true)); label("$n$",Np+(+0.12,0.07)); draw(anglemark2(R,M,D,13,true)); label("$m$",M+(+0.25,0.03)); draw(anglemark2(M,D,P,29)); label("$d$",D+(-0.75,0.095)); label("$R$",R,N,f); label("$M$",M+(-.07,.07),f); label("$N$",Np+(-.08,.15),f); label("$P$",P,S,f); label("$S$",Sp,S,f); label("$Q$",Q,S,f); label("$D$",D,S,f);[/asy] Looking at triangle PRQ, we have $\angle RPD+\angle RQS+\angle MRN=180$ and from the given statement $\angle NMR=\frac{1}{2}\angle MRN$, so looking at triangle MOR $\angle NMR=90-\frac{\angle RPD+\angle RQS}{2}$, which rules out